0
大学数学基礎解説
文献あり

ABJ anomaly: オペレータ形式におけるpoint splittingの方法

41
0
$$\newcommand{all}[1]{\left\langle#1\right\rangle} \newcommand{blr}[1]{\left[#1\right]} \newcommand{car}[1]{\left\{#1\right\}} \newcommand{di}[0]{\displaystyle} \newcommand{fr}[2]{\frac{#1}{#2}} \newcommand{lr}[1]{\left(#1\right)} \newcommand{ma}[1]{\(\di{#1}\)} $$

本記事ではオペレータ積の形で軸性ベクトルカレントを扱い、正則化をpoint splittingで行うことでABJ anomalyを導きます。

参考文献はRef.Peskinです。というかその日本語訳+$\alpha$という感じです。

Point splittingによる正則化

軸性ベクトルカレントは通常次のように書きます:
\begin{align} j^{\mu 5}=\bar\psi(x)\gamma^\mu\gamma_5\psi(x) \end{align}
ここで$\psi,\bar\psi$はオペレータとします。しかしこの表式はwell definedではありません。なぜなら同一点のオペレータの積は発散を含むからです。そこで$\bar\psi$$x^\mu+\epsilon^\mu/2$に、$\psi$$x^\mu-\epsilon^\mu/2$に置くことで正則化します。ただしこれを行うとgauge invarianceが失われるため、$\bar\psi(x+\epsilon/2)$$\psi(x-\epsilon/2)$の間をWilson line
\begin{align} \exp\left[ -ie\int_{x-\epsilon/2}^{x+\epsilon/2} dz^\mu A_\mu(z) \right] \end{align}
で結びます。このように定義した$j^{\mu 5}$の発散は以下のようになります:
\begin{align} \partial_\mu j^{\mu 5} =\underset{\epsilon\rightarrow 0}{\text{symm lim}} \ \partial_\mu \left\{ \bar\psi(x+\epsilon/2)\gamma^\mu\gamma_5 \exp\left[ -ie\int_{x-\epsilon/2}^{x+\epsilon/2} dz^\mu A_\mu(x) \right] \psi(x-\epsilon/2) \right\}\tag{1} \end{align}
ここで$\text{symm lim}$は、$\epsilon^\mu\rightarrow 0$を以下を満たすようにとることを意味します:
\begin{align} \underset{\epsilon\rightarrow 0}{\text{symm lim}} \ \frac{\epsilon^\mu \epsilon^\nu}{\epsilon^2}=\frac{1}{d}g^{\mu\nu} \ \ \ (d:\text{dimension}) \end{align}
これはすなわち$\epsilon^\mu$を回転対称性を保つようにゼロに近づけるということです。

Eq.(1)を計算していきます:
\begin{align} \partial_\mu j^{\mu 5} &=\underset{\epsilon\rightarrow 0}{\text{symm lim}} \ \partial_\mu \left\{ \bar\psi(x+\epsilon/2)\gamma^\mu\gamma_5 \exp\left[ -ie\int_{x-\epsilon/2}^{x+\epsilon/2} dz^\mu A_\mu(x) \right] \psi(x-\epsilon/2) \right\}\\ &= \underset{\epsilon\rightarrow 0}{\text{symm lim}} \ \Bigg\{ \partial_\mu\bar\psi(x+\epsilon/2)\gamma^\mu\gamma_5 \exp\left[ -ie\int_{x-\epsilon/2}^{x+\epsilon/2} dz^\mu A_\mu(x) \right] \psi(x-\epsilon/2) \\ &\hspace{3.5cm}+ \bar\psi(x+\epsilon/2)\gamma^\mu\gamma_5 \exp\left[ -ie\int_{x-\epsilon/2}^{x+\epsilon/2} dz^\mu A_\mu(x) \right] \partial_\mu\psi(x-\epsilon/2)\\ &\hspace{3.5cm}+ \bar\psi(x+\epsilon/2)\gamma^\mu\gamma_5 \left[ -ie\epsilon^\nu\partial_\mu A_\nu(x) \right] \psi(x-\epsilon/2) \Bigg\} \end{align}
EoM:$\not \partial\psi=-ie\not A\psi,\ \partial_\mu\bar\psi\gamma^\mu=ie\bar\psi\not A$を用いれば
\begin{align} &=\underset{\epsilon\rightarrow 0}{\text{symm lim}} \ \Bigg\{ ie\bar\psi(x+\epsilon/2) \not A(x+\epsilon/2)\gamma_5 \exp\left[ -ie\int_{x-\epsilon/2}^{x+\epsilon/2} dz^\mu A_\mu(x) \right] \psi(x-\epsilon/2) \\ &\hspace{3.5cm}+ \bar\psi(x+\epsilon/2)\gamma_5 \exp\left[ -ie\int_{x-\epsilon/2}^{x+\epsilon/2} dz^\mu A_\mu(x) \right] (ie)\not A(x-\epsilon/2)\psi(x-\epsilon/2)\\ &\hspace{3.5cm}+ \bar\psi(x+\epsilon/2)\gamma^\mu\gamma_5 \left[ -ie\epsilon^\nu\partial_\mu A_\nu(x) \right] \psi(x-\epsilon/2) \Bigg\}\\ &\sim \underset{\epsilon\rightarrow 0}{\text{symm lim}} \ \Bigg\{ ie\bar\psi(x+\epsilon/2)\not A(x+\epsilon/2) \gamma^5\psi(x-\epsilon/2) +\bar\psi(x+\epsilon/2)\gamma^5(ie)\not A(x-\epsilon/2)\psi(x-\epsilon/2)\\ &\hspace{3.5cm}+\bar\psi(x+\epsilon/2)\gamma^\mu\gamma^5 (-ie\epsilon^\nu\partial_\mu A_\nu(x)) \psi(x-\epsilon/2) \Bigg\}\\ &=\underset{\epsilon\rightarrow 0}{\text{symm lim}} \ \left\{ \bar\psi(x+\epsilon/2)(ie\epsilon^\mu\gamma^\nu\partial_\mu A_\nu -ie\epsilon^\nu\gamma^\mu\partial_\mu A_\nu) \gamma^5\psi(x-\epsilon/2) \right\}\\ &=\underset{\epsilon\rightarrow 0}{\text{symm lim}} \ \left\{ \bar\psi(x+\epsilon/2) [-ie\gamma^\mu\epsilon^\nu(\partial_\mu A_\nu-\partial_\nu A_\mu)] \gamma^5\psi(x-\epsilon/2) \right\}\tag{2} \end{align}
これは一見$\epsilon\rightarrow 0$でゼロにみえますが、$\bar\psi(x+\epsilon/2)\gamma^\mu\gamma^5\psi(x-\epsilon/2)$の期待値がこの極限で発散するため有限になります。

4次元において、background field $A_\mu$が存在するとき、fermionの伝播の最低次は図1のダイアグラムです。これを計算すると

Fermion伝播の最低次のダイアグラム Fermion伝播の最低次のダイアグラム

\begin{align} \psi(y)^\bullet\bar\psi(z)^\bullet&= \int\frac{d^4k}{(2\pi)^4}e^{-ik\cdot(y-z)}\frac{i\not k}{k^2}\\ &=-\not \partial\int\frac{d^4k}{(2\pi)^4}e^{-ik\cdot(y-z)}\frac{1}{k^2}\\ &=-\not \partial \left(\frac{i}{4\pi^2}\frac{1}{(y-z)^2}\right)\\ &=\frac{-i}{2\pi^2}\frac{\gamma^\alpha(y-z)_\alpha}{(y-z)^4} \end{align}
となります($A^\bullet B^\bullet$は演算子$A$$B$のWick contractionを表す)。これは$\gamma^\mu\gamma^5$とともにtrをとるとゼロになるので寄与しません。back ground field $A_\mu$の寄与を1つ含むfermionのpropagation(図2。$\otimes$$A_\mu$を表す)は以下のように計算できます:

Fermion伝播の!FORMULA[30][1207907657][0]のダイアグラム Fermion伝播の${e^2}$のダイアグラム

\begin{align} \int\frac{d^4k}{(2\pi)^4}\frac{d^4p}{(2\pi)^4} e^{-i(k+p)\cdot y}e^{ik\cdot z} \frac{i(\not k+\not p)}{(k+p)^2} (-ie\not A(p))\frac{i\not k}{k^2} \end{align}
よってこのダイアグラムによる$\bar\psi(x+\epsilon/2) \gamma^\mu\gamma^5\psi(x-\epsilon/2)$への寄与は以下のようになります:
\begin{align} &\langle\bar\psi(x+\epsilon/2) \gamma^\mu\gamma^5\psi(x-\epsilon/2) \rangle|_{\text{図2}}\\ &= \int\frac{d^4k}{(2\pi)^4}\frac{d^4p}{(2\pi)^4} e^{-i(k+p)\cdot (x-\epsilon/2)}e^{ik\cdot (x+\epsilon/2)} {\rm tr} \left[ -i\gamma^\mu\gamma^5 \frac{i(\not k+\not p)}{(k+p)^2} (-ie\not A(p))\frac{i\not k}{k^2} \right]\\ &=\int\frac{d^4k}{(2\pi)^4}\frac{d^4p}{(2\pi)^4} e^{i\epsilon\cdot k}e^{-ip\cdot x}e^{ip\cdot \epsilon/2} \frac{4e\epsilon^{\mu\alpha\beta\gamma}(k+p)_\alpha A_\beta(p)k_\gamma}{(k+p)^2k^2} \end{align}
分子で$k$が2つかかるものはtrで消えます。また被積分関数の$k$が大きな部分が$\epsilon\rightarrow 0$の極限で発散に寄与するので
\begin{align} &\rightarrow \int\frac{d^4k}{(2\pi)^4}\frac{d^4p}{(2\pi)^4} e^{i\epsilon\cdot k}e^{-ip\cdot x}e^{ip\cdot \epsilon/2} \frac{4e\epsilon^{\mu\alpha\beta\gamma}p_\alpha A_\beta(p)k_\gamma}{k^4}\\ &=4e\epsilon^{\mu\lambda\beta\gamma}\int\frac{d^4p}{(2\pi)^4}e^{-ip\cdot x}e^{ip\cdot \epsilon/2} p_\alpha A_\beta(p) i\frac{\partial}{\partial \epsilon^\gamma}\int\frac{d^4k}{(2\pi)^4}e^{i\epsilon\cdot k}\frac{1}{k^4} \end{align}
ここで
\begin{align} \partial_\alpha A_\beta(x)=\int\frac{d^4p}{(2\pi)^4}(-ip_\alpha)e^{-ip\cdot x}A_\beta(p) \end{align}
であり、$e^{ip\cdot \epsilon/2}$は無視できるので
\begin{align} =4e\epsilon^{\mu\lambda\beta\gamma} (i\partial_\alpha A_\beta(x)) i\frac{\partial}{\partial \epsilon^\gamma}\int\frac{d^4k}{(2\pi)^4}e^{i\epsilon\cdot k}\frac{1}{k^4} \end{align}
Appendixの計算より
\begin{align} \int \frac{d^4k}{(2\pi)^4}e^{i\epsilon\cdot k}\frac{1}{k^4} \xrightarrow{\text{Euclid化}} i\int \frac{d^4k}{(2\pi)^4}e^{-i\epsilon\cdot k}\frac{1}{k^4} =\frac{i}{16\pi^2}\log\frac{1}{\epsilon^2}\tag{3} \end{align}
となります。ゆえにEq.(2)は
\begin{align} \langle\bar\psi(x+\epsilon/2) \gamma^\mu\gamma^5\psi(x-\epsilon/2)\rangle &=4e\epsilon^{\mu\lambda\beta\gamma}\int\frac{d^4p}{(2\pi)^4}e^{-ip\cdot x}e^{ip\cdot \epsilon/2} p_\alpha A_\beta(p) i\frac{\partial}{\partial \epsilon^\gamma}\int\frac{d^4k}{(2\pi)^4}e^{i\epsilon\cdot k}\frac{1}{k^4} \\ &=4e\epsilon^{\mu\alpha\eta\gamma}(i\partial_\alpha A_\beta(x))i\frac{\partial}{\partial\epsilon^\alpha} \left( \frac{i}{16\pi^2}\log\frac{1}{\epsilon^2} \right)\\ &=2e\epsilon^{\alpha\beta\mu\gamma}F_{\alpha\beta}(x) \left(-\frac{i}{8\pi^2}\frac{\epsilon_\gamma}{\epsilon^2}\right) \end{align}
となります。これはsymmetric limitで$1/\epsilon$で発散します。よってEq.(2)の$\epsilon^\nu$と積をとることで有限の寄与を与えます。計算すれば
\begin{align} \partial_\mu j^{5\mu}&= \underset{\epsilon\rightarrow 0}{\text{symm lim}} \ \left\{ \bar\psi(x+\epsilon/2) [-ie\gamma^\mu\epsilon^\nu(\partial_\mu A_\nu-\partial_\nu A_\mu)] \gamma^5\psi(x-\epsilon/2) \right\}\\ &=\underset{\epsilon\rightarrow 0}{\text{symm lim}} \ \left\{ (-ie\epsilon^\nu F_{\mu\nu}(x)) 2e\epsilon^{\alpha\beta\mu\gamma}F_{\alpha\beta}(x) \left(-\frac{i}{8\pi^2}\frac{\epsilon_\gamma}{\epsilon^2}\right) \right\}\\ &=-\frac{e^2}{4\pi^2}\epsilon^{\alpha\beta\mu\gamma} F_{\alpha\beta}F_{\mu\nu} \left( \underset{\epsilon\rightarrow 0}{\text{symm lim}} \ \frac{\epsilon_\gamma\epsilon^\nu}{\epsilon^2} \right)\\ &=-\frac{e^2}{16\pi^2}\epsilon^{\alpha\beta\mu\nu}F_{\alpha\beta}F_{\mu\nu} \end{align}
を得ます。これはこれまで計算してきた結果と一致します。

おしまい。${}_\blacksquare$



Appendix: Eq.(3)の導出

(※数学的には厳密でない議論かと思いますがご容赦ください)

$\displaystyle \int \frac{d^4k}{(2\pi)^4}e^{i\epsilon\cdot k}\frac{1}{k^4}$を計算します。まずEuclid化します:
\begin{align} \int \frac{d^4k}{(2\pi)^4}e^{i\epsilon\cdot k}\frac{1}{k^4} \xrightarrow{\text{Euclid化}} i\int \frac{d^4k}{(2\pi)^4}e^{-i\epsilon\cdot k}\frac{1}{k^4} \end{align}
極座標に移り書き直せば
\begin{align} =\frac{4\pi i}{(2\pi)^4} \int_0^\pi d\theta \sin^2\theta \int_0^\infty \frac{1}{k}e^{-ik\epsilon\cos\theta}dk\tag{4} \end{align}
ただし$k=|k^\mu|, \epsilon=|\epsilon^\mu|$です。$\theta$$\epsilon^\mu$$k^\mu$のなす角です。$k$の積分でnaiveに$k$を変数変換すると$\epsilon$の寄与は消えます。しかしこの積分はinfraredで発散しています(UVは$\epsilon>0$ならば$\epsilon$に依存しない有限の寄与)。そこで、発散を取り除くのと、$\epsilon$に依存した部分のみを取り出すために、$k$積分を
\begin{align} \int_0^\infty\frac{1}{k}e^{-ik\epsilon\cos\theta}dk \rightarrow \int d\epsilon\frac{\partial}{\partial\epsilon}\int_0^\infty\frac{1}{k}e^{-ik\epsilon\cos\theta}dk \end{align}
とします(こういう正則化を採用したと思ってください)。計算を進めると
\begin{align} \int d\epsilon\frac{\partial}{\partial\epsilon}\int_0^\infty\frac{1}{k}e^{-ik\epsilon\cos\theta}dk &=\int d\epsilon \left[ \int_0^\infty (-i\cos\theta)e^{-ik\epsilon\cos\theta}dk \right]\\ &=\int d\epsilon \frac{1}{\epsilon} \left[ e^{-ik\epsilon\cos\theta} \right]_0^\infty \end{align}
ここで積分の上端の無限大の寄与は無視します($k$の積分経路を実軸から少しだけ傾けてdamping factorを導入したと思っても良い)。すると
\begin{align} =-\log\epsilon \end{align}
となります。以上より
\begin{align} \int \frac{d^4k}{(2\pi)^4}e^{i\epsilon\cdot k}\frac{1}{k^4} &= {\rm Eq.}(4)\\ &=\frac{4\pi i}{(2\pi)^4} \int_0^\pi d\theta \sin^2\theta (-\log\epsilon)\\ &=\frac{4\pi i}{(2\pi)^4}\frac{\pi}{2}(-\log\epsilon)\\ &=\frac{i}{16\pi^2}\log\frac{1}{\epsilon^2} \end{align}
を得ます。${}_\blacksquare$

参考文献

[1]
Michael E. Peskin, Daniel V. Schroeder, An Introduction to Quantum Field Theory, Westview Press, 1995, P655-
投稿日:2023714

この記事を高評価した人

高評価したユーザはいません

この記事に送られたバッジ

バッジはありません。

投稿者

bisaitama
bisaitama
80
38878

コメント

他の人のコメント

コメントはありません。
読み込み中...
読み込み中